0 Daumen
437 Aufrufe

Die Aufgabe lautet: Berechnen Sie (1+a)^k+(1-a)^k, wobei a := i (komplexe Zahl) und k Element von N

Wie kommt man auf eine sinnvolle Lösung?


Ich habe mir ein Programm geschrieben, was (1+a)^k+(1-a)^k , mit k = {0,1,2,...,100} berechnet. Dabei konnte ich feststellen, dass bei der Addition der beiden "Terme" (sagt man dazu Terme, denke nicht oder?:( ) der Imaginärteil entfällt.

Weiter konnte ich feststellen, dass nach jeden vierten k, (1+a)^k+(1-a)^k = 0 ergibt und sich da Vorzeichen der Lösung danach ändert.

Des Weiteren kann man feststellen, dass manche Lösungen der geraden k's mit den ungeraden k's übereinstimmen.

Könnte man hier etwas mit einer Fallunterscheidung machen? Wenn ja, was? Leider sehe ich anscheinend das offensichtliche nicht:)


Avatar von

2 Antworten

0 Daumen

(1+i)k+(1-i)k

k=1

Zeichne die Zeiger 1+i und 1-i in der komplexen Zahlenebene ein. (Argumente + und - π/4 = 45° und Beträge √(1+1) = √2. 

Sie sind symmetrisch zur reellen Achse. 

Bei der Addition (wie Vektoraddition mit Parallelogrammregel) entfällt der imaginäre Anteil. Es resultiert eine positive reelle Zahl. 

1+i + 1 - i = 2

k=2 

Nun kommen die geometrischen Eigenschaften der Multiplikation (Argumente addieren und Beträge multiplizieren) ins Spiel:

(1+i)^2 = (1+i)(1+i) hat Argument 90° und Betrag 2.

(1-i)^2 hat Argument -90° und Betrag 2.

Insgesamt ergibt sich 0.


(1+i)^2+(1-i)^2 = 0 

k=3 

(1+i)^3  Argument 135° Betrag 2*√(2)

(1-i)^3 Argument -135° Betrag 2*√(2)

Es ergibt sich durch vektorielle Addition

(1+i)^3+(1-i)^3 = - 4 

k=4

(1+i)^4  Argument 180°, Betrag 4

(1-i)4  Argument -80°, Betrag 4

(1+i)^4+(1-i)^4 = - 8

k=5

(1+i)^5 Argument -135°, Betrag 4*√2

+(1-i)^5 Argument 135°, Betrag 4*√2 

(1+i)^5+(1-i)^5 = - 8 

k=6 

(1+i)^6+(1-i)^6 = 0 

usw, 

Ich hoffe, ich habe mich nun nicht verrechnet. 

Du kannst eine Fallunterscheidung mit 4 Fällen machen:

k=0, k=1, k=2, k=3 modulo 4.

So haben die Resultate immer wieder dieselben Argumente und du brauchst dich nur noch um die Beträge zu kümmern und die in Abhängigkeit von k auszudrücken. 


Avatar von 7,6 k
0 Daumen

Vielleicht hilft

1 + i = √2  *  e i*π/4     und  1 - i = √2  *  e -i*π/4

Dann sind die Potenzen schon mal

(1 + i )k= (√2)k  *  e i*π*k/4     und  (1 - i )k= (√2)k  *  e -i*π*k/4

also (1 + i )k   +  (1 - i )k

= (√2)k  *  e i*π*k/4   + (√2)k  *  e -i*π*k/4

= (√2)k  * ( e i*π*k/4   +  e -i*π*k/4  )

=(√2)k  * ( cos( π*k/4 ) + sin (π*k/4 ) + cos( -π*k/4 ) + sin (-π*k/4 ))

=(√2)k  * ( cos( π*k/4 ) + sin (π*k/4 ) + cos( π*k/4 ) - sin (-π*k/4 ))

=(√2)k  * 2*cos( π*k/4 )

Avatar von 288 k 🚀

Ein anderes Problem?

Stell deine Frage

Willkommen bei der Mathelounge! Stell deine Frage einfach und kostenlos

x
Made by a lovely community